LSAT and Law School Admissions Forum

Get expert LSAT preparation and law school admissions advice from PowerScore Test Preparation.

 Administrator
PowerScore Staff
  • PowerScore Staff
  • Posts: 8917
  • Joined: Feb 02, 2011
|
#81462
Complete Question Explanation

Resolve the Paradox. The correct answer choice is (D).

Answer choice (A):

Answer choice (B):

Answer choice (C):

Answer choice (D): This is the correct answer choice.

Answer choice (E):

This explanation is still in progress. Please post any questions below!
 melissa7t
  • Posts: 2
  • Joined: Jan 08, 2019
|
#61714
I am having trouble understanding why answer choice B is incorrect and why D is correct.
 Robert Carroll
PowerScore Staff
  • PowerScore Staff
  • Posts: 1787
  • Joined: Dec 06, 2013
|
#61730
Melissa,

The paradox in the stimulus is that one would expect melanoma cases to decline if people are taking the better advice on avoiding the sun. Answer choice (D) addresses this by showing that the melanoma cases might not have increased in 1982, because better diagnostic methods could detect more cases than before. Thus, the cases aren't increasing; detection methods are resulting in more DETECTED cases.

Answer choice (B) does not address the paradox because we already know that people were more likely to have unhealthy practices relating to the sun before 1980, when the better advice came out. This would help explain melanoma cases before 1980; it can't explain the higher reported cases in 1982.

Robert Carroll

Get the most out of your LSAT Prep Plus subscription.

Analyze and track your performance with our Testing and Analytics Package.